stetigkeit zeigen

Neue Frage »

timmy Auf diesen Beitrag antworten »
stetigkeit zeigen
hallo!!!

ich bin schon am verzweifeln weil ich diese stetigkeit einfach nicht verstehe. oder eher gesagt ich kann sich nicht nachweisen.

habe drei kurze aufgaben bekommen die ich machen muss aber komme bei keiner der weiter.

1. ich muss mit dem delta-epsilon-kriterium zeigen, dass f(x)=x^3 stetig ist.

2. f auf stetigkeit in x=4 untersuchen wobei dann f(x):=x+1 für und für x>4 ist.

3. Wie muss ich das y wählen, damit f in x=3 stetig ist. für und y für x=3.


wie gesagt hab schon hin und her probiert. aber komm einfach nicht klar mit diesen aufgaben.
wär toll wenn mir da jemand bei helfen könnte.
therisen Auf diesen Beitrag antworten »

Hallo,

zu 1)
Betrachte


zu 2) Wenn f in stetig sein soll, müssen dort links- und rechtsseitiger Grenzwert übereinstimmen.


zu 3) Schreibe und wende den Trick aus 1) an. Anschließend kürzen und für x den Wert 3 einsetzen. Das ist dein y.


Gruß, therisen
timmy Auf diesen Beitrag antworten »

gut bei 1 muss ich dann zeigen ,






Zitat:
Original von therisen



ich versteh noch nicht ganz, warum das kleiner als epsilon ist??
timmy Auf diesen Beitrag antworten »

bei zwei hab ich lim(x+1)=5 und lim(x^2-1)=15, damit sind sie nicht gleich und f ist an der stelle x=4 nicht stetig. ich hab das mit nem programm berechnen lasse, da ich leider nicht weiss wie man das per hand macht. könnte mir da jemand einen tipp geben?

ich hab mir gedacht erst die ableitung berechnen, und dann x=4 einsetzten. aber dann kommen ganz andere werte raus verwirrt
therisen Auf diesen Beitrag antworten »

Hi,

ich hatte mich oben verlesen, ich dachte es ginge um die Funktion f(x)=1/x^3 Big Laugh Das macht aber nichts, denn den Trick kannst du ja für f(x)=x^3 auch anwenden.

Der Term SOLL kleiner als werden.


Gruß, therisen
timmy Auf diesen Beitrag antworten »

ja aber man muss doch eigentlich ein delta-epsilon angeben, oder nicht?

wir hatte das mit f(x)=x^2 gemacht und zwar so










wobei d=Delta und dann haben wir das delta konkter angegeben


so sollten wirs auch mit x^3 machen, und das ist genau mein problem es so hinzukriegen.
 
 
timmy Auf diesen Beitrag antworten »

@therisen
hab jetzt in 3 das so gemacht wie du gesagt hast und da kommt y=9 raus, kann das so etwa hinkommen? hoffe ich hatte dich richtig verstanden.

wäre nett wenn mir nochmal jemand kurz sagen könnte, wie man den recht- und linksseitigen grenzwert berechnet. verwirrt
therisen Auf diesen Beitrag antworten »

Hi,

zu 3) Es muss herauskommen. Zeig mal deine Rechenschritte!

zu 2)




zu 1) Du hast schon ganz gut angefangen:



Dieser Term soll nun kleiner als ein vorgegebenes werden. Dabei müssen wir das entsprechend wählen.

Für gilt


Überlege dir nun, wie du dein wählen könntest.

Gruß, therisen
timmy Auf diesen Beitrag antworten »

ja klar, bei mir kommt auch 27 raus. verrechnet ich trottel Augenzwinkern

2 ist jetzt auch klar, danke!!

3. schaun ich mir jetzt nochmal genau an.
timmy Auf diesen Beitrag antworten »

hi, hab mir das jetzt mit der 3. versucht zu überlegen.

Zitat:
Original von therisen
gilt


Warum gilt das?
Und wie kann ich das delta-epsilon mit hilfe dieser Ungeleichung bestimmen betimmen? verwirrt
timmy Auf diesen Beitrag antworten »

hi nochmal, ich hab hier mal so ne idee

und zwar gilt ja =>

dann kann man das doch gleich so aufschreiben





und wenn man das jetzt nach Delta auflöst ist man doch schon ferig!!
therisen Auf diesen Beitrag antworten »

Zitat:
Original von timmy
Zitat:
Original von therisen
gilt


Warum gilt das?
Und wie kann ich das delta-epsilon mit hilfe dieser Ungeleichung bestimmen betimmen? verwirrt


Ersteres hast du dir eigentlich schon selber beantwortet. Aus folgt nämlich . Damit erhältst du

Also folgt aus , dass gilt:


Gruß, therisen
timmy Auf diesen Beitrag antworten »

ja gut, wenn mans so macht versteh ichs schon.

nur noch eine frage dazu, darf man einfach annehmen, dass |x-x_0|<1 ist? damit haben wir doch das delta einfach festgelegt?
therisen Auf diesen Beitrag antworten »

Zitat:
Original von timmy
nur noch eine frage dazu, darf man einfach annehmen, dass |x-x_0|<1 ist? damit haben wir doch das delta einfach festgelegt?



Das ist ja schließlich gesucht, d.h. wir sollen ihm einen Wert zuweisen. Warum also nicht höchstens den Wert 1? Im Folgenden zeigt sich ja, dass es funktioniert!
Gilt , so auch trivialerweise . Ansonsten ist und dann gilt diese Ungleichung erst recht. In jedem Fall können wir also durch abschätzen und in jedem Fall auch .



Gruß, therisen
Neue Frage »
Antworten »



Verwandte Themen

Die Beliebtesten »
Die Größten »
Die Neuesten »